¿La ecuación de Peskin y Schroeder. (4.26), U(t1,t2)U(t2,t3)=U(t1,t3)U(t1,t2)U(t2,t3)=U(t1,t3)U(t_1,t_2)U( t_2,t_3) = U(t_1,t_3) implica [H0,Pista]=0[H0,Pista]=0[H_0,H_{int}] = 0?

La ecuación de Peskin y Schroeder (4.17) define el operador,

(4.17) tu ( t , t 0 )   =   mi i ( t t 0 ) H 0 mi i ( t t 0 ) H
dónde
(4.12) H   =   H 0 + H En t
es el hamiltoniano completo y H 0 es el Hamilton libre, ambos en el cuadro de Schrödinger. En la ecuación (4.26), Peskin y Schroeder establecen que el operador satisface la siguiente identidad,
(4.26) tu ( t 1 , t 2 ) tu ( t 2 , t 3 )   =   tu ( t 1 , t 3 )
dónde t 1 t 2 t 3 . ¿Implica esto que el hamiltoniano libre conmuta con la interacción
[ H 0 , H En t ]   =   0   ?
Aquí está mi argumento de que sí.

en la condición t 1 t 2 t 3 llevar t 2 = 0 . La identidad es entonces,

tu ( t 1 , 0 ) tu ( 0 , t 3 ) = tu ( t 1 , t 3 )   .
Sustituye la definición,
mi i t 1 H 0 mi i t 1 H mi i t 3 H 0 mi i t 3 H = mi i ( t 1 t 3 ) H 0 mi i ( t 1 t 3 ) H
y simplificar para obtener,
mi i t 1 H mi i t 3 H 0 = mi i t 3 H 0 mi i t 1 H
con t 1 0 t 3 . Poner t 1 = t y t 3 = t .
mi i t H mi i t H 0 = mi i t H 0 mi i t H
Expandiendo a segundo orden en t ,
( 1 i t H t 2 2 H H ) ( 1 + i t H 0 t 2 2 H 0 H 0 ) = ( 1 + i t H 0 t 2 2 H 0 H 0 ) ( 1 i t H t 2 2 H H )
da como resultado,
H H 0 = H 0 H
de modo que [ H 0 , H ] = 0 . Ahora H = H 0 + H i norte t por lo que el hamiltoniano libre debe conmutar con la interacción.
[ H 0 , H i norte t ] = 0
En Peskin y Schroeder, el contexto de este material es el campo escalar que interactúa con el hamiltoniano,
H = d 3 X ( 1 2 π ( t , X ) 2 + 1 2 ϕ X r ϕ X r + V ( ϕ ) )   .
En la teoría clásica, el PB es,
[ H 0 , H i norte t ] PAG B = d 3 X d H 0 d π d H i norte t d ϕ = d 3 X   π d V d ϕ = d d t d 3 X   V ( ϕ ( t , X ) )
Pasando a la teoría cuántica,
[ H 0 , H i norte t ] = i d d t d 3 X   V ( ϕ ( t , X ) )
de modo que [ H 0 , H i norte t ] = 0   implica la integral de V ( ϕ ) es una carga conservada; ¿Es este también un resultado correcto?

No hay tal requisito en H 0 y H i norte t . Debe ser un error tipográfico --- la definición correcta del operador de evolución en la imagen de interacción es mi i H t conjugado por mi i H 0 t .
Creo que es un error tipográfico.
Esto ya falla con simple massless ϕ 4 teoría. Claramente, ϕ 4 no puede viajar con Π ϕ , y por lo tanto no conmutará con el hamiltoniano.
"¿Esto implica que el hamiltoniano libre conmuta con la interacción [H0,Hint]=0 ?" No, no lo hace.

Respuestas (3)

Árbitro. 1 escribe la fórmula correcta

(4.25) tu ( t , t )   =   mi i H 0 ( t t 0 ) mi i H ( t t ) mi i H 0 ( t t 0 ) , t     t ,

que satisface

(4.26) tu ( t 1 , t 2 ) tu ( t 2 , t 3 )   =   tu ( t 1 , t 3 ) , t 1     t 2     t 3 .

Aquí t 0 es un instante inicial fiduciario arbitrario pero fijo donde concuerdan los operadores y estados en la imagen de Schrödinger , la imagen de Heisenberg y la imagen de interacción . Para t t 0 , los tres cuadros ya no son los mismos, aunque siguen siendo unitario equivalente.

Para t = t 0 , ecuación (4.25) se simplifica a

(4.17) tu ( t , t 0 )   =   mi i H 0 ( t t 0 ) mi i H ( t t 0 ) .

Parece que OP reemplaza por error t 0 en la ec. (4.17) con un tiempo arbitrario t t . La ecuación resultante

tu ( t , t )   =   mi i H 0 ( t t ) mi i H ( t t ) . ( ¡Equivocado! )

no es correcto

Referencias:

  1. ME Peskin y DV Schroeder, Introducción a QFT; Sección 4.2.

El hecho de que

(1) tu I ( t 1 , t 2 ) tu I ( t 2 , t 3 ) = tu I ( t 1 , t 3 )
en la imagen de interacción no se basa en H 0 y H En t desplazamiento, pero se puede derivar sin esa suposición de la ecuación de Tomonaga-Schwinger
i t tu I ( t , t 0 ) = H I ( t ) tu I ( t , t 0 )
con H I ( t ) := mi i H 0 t H En t mi i H 0 t como H En t evolucionado por H 0 y que esa ecuación tiene solución en serie de Dyson
tu I ( t , t 0 ) = T Exp ( i t 0 t H I ( t ) d t )
a partir del cual ( 1 ) se puede mostrar

la versión de tu I dado por Peskin-Schröder (y algunos otros que probablemente copiaron de ellos) es incorrecto, la versión correcta es

tu I ( t , t 0 ) = mi i H 0 t mi i H ( t t 0 ) mi i H 0 t 0
que se sigue de ψ I ( t ) := mi i H 0 t ψ S ( t ) y ψ I ( t ) = tu I ( t , t 0 ) ψ I ( t 0 ) , dónde S denota estados de Schrödinger. Esto obviamente también cumple ( 1 ) sin suposiciones adicionales.

Primero, tomó las expansiones de Taylor de segundo orden de mi i t H y mi i t H 0 , entonces, si sus siguientes cálculos fueron correctos, H y H 0 conmutarían entre sí solo al segundo orden de tiempo t , no para todo orden de t (o simplemente viaje aproximadamente).

Segundo, para 4 operadores A , B , H , k :

A H B = A k B A ( H k ) B = 0.
De esta relación podemos derivar H = k si (y solo si) para todo A, B (H, K permanecen sin cambios), la relación anterior siempre se cumple. En sus cálculos,
A = mi i t 1 H 0 , B = mi i t 3 H , H = mi i t 1 H mi i t 3 H 0 , k = mi i t 3 H 0 mi i t 1 H
Vemos eso, A y k aquí están relacionados entre sí (también para B y H ). Entonces, de la condición anterior, no podemos implicar H = k .

Por lo tanto, su resultado no es correcto.

A y B tener inversas. Entonces, a partir de A H B = A k B , A 1 A H B B 1 = A 1 A k B B 1 implica H = k .
Además, para su primer punto, la igualdad de la serie de potencias se cumple para todos t , y no hay otros términos de segundo orden de las expansiones de orden superior. Como, para dos series convergentes, norte a norte t norte = norte b norte t norte t si y solo si a norte = b norte , el argumento en el OP sería correcto (usa a 2 = b 2 ), si la forma dada de tu I eran correctos.